Inequality $sumlimits_{cyc}frac{a^3}{13a^2+5b^2}geqfrac{a+b+c}{18}$











up vote
25
down vote

favorite
24













Let $a$, $b$ and $c$ be positive numbers. Prove that:
$$frac{a^3}{13a^2+5b^2}+frac{b^3}{13b^2+5c^2}+frac{c^3}{13c^2+5a^2}geqfrac{a+b+c}{18}$$




This inequality is strengthening of the following Vasile Cirtoaje's one, which he created in 2005.




Let $a$, $b$ and $c$ be positive numbers. Prove that:
$$frac{a^3}{2a^2+b^2}+frac{b^3}{2b^2+c^2}+frac{c^3}{2c^2+a^2}geqfrac{a+b+c}{3}.$$




My proof of this inequality you can see here: https://artofproblemsolving.com/community/c6h22937p427220



But this way does not help for the starting inequality.



A big problem we have around the point $(a,b,c)=(0.785, 1.25, 1.861)$ because the difference between the LHS and the RHS in this point is $0.0000158...$.



I tried also to use Cauchy-Schwarz, but without success.



Also, I think the BW (see here https://math.stackexchange.com/tags/buffalo-way/info I tryed!) does not help.










share|cite|improve this question




















  • 2




    Is the function $f : (a,b,c) rightarrow sumlimits_{cyc} frac{6a^3}{13a^2+5b^2} $ concave? Also, C-S = "Cauchy-Schwarz inequality"? Avoid this kind of unconventional abbreviation, please.
    – Vincent
    May 8 '16 at 18:17












  • See also math.stackexchange.com/questions/1775572/…
    – David Quinn
    May 8 '16 at 18:42






  • 1




    @Santropedro There is the following known inequality $sumlimits_{cyc}frac{a^3}{a^2+b^2}geqfrac{a+b+c}{2}$. I think, it's just interesting, for which maximal value of $k$ the following inequality is still true. $sumlimits_{cyc}frac{a^3}{ka^2+b^2}geqfrac{a+b+c}{k+1}$.
    – Michael Rozenberg
    Apr 19 '17 at 17:28






  • 1




    @Santropedro The value $,k=2.6,$ in the OP is pretty sharp. I found that $k=2.603279$ is already beyond: Then Michael's given $(0.785, 1.25, 1.861)$ violates the ineq..
    – Hanno
    May 11 '17 at 18:24






  • 1




    Further result obtained via random-based simulation: $(0.785, 1.2535, 1.873)$ doesn't satisfy the inequality if $,k=2.603262,$. Below that $k$-value I didn't detect a violation.
    – Hanno
    May 11 '17 at 18:50















up vote
25
down vote

favorite
24













Let $a$, $b$ and $c$ be positive numbers. Prove that:
$$frac{a^3}{13a^2+5b^2}+frac{b^3}{13b^2+5c^2}+frac{c^3}{13c^2+5a^2}geqfrac{a+b+c}{18}$$




This inequality is strengthening of the following Vasile Cirtoaje's one, which he created in 2005.




Let $a$, $b$ and $c$ be positive numbers. Prove that:
$$frac{a^3}{2a^2+b^2}+frac{b^3}{2b^2+c^2}+frac{c^3}{2c^2+a^2}geqfrac{a+b+c}{3}.$$




My proof of this inequality you can see here: https://artofproblemsolving.com/community/c6h22937p427220



But this way does not help for the starting inequality.



A big problem we have around the point $(a,b,c)=(0.785, 1.25, 1.861)$ because the difference between the LHS and the RHS in this point is $0.0000158...$.



I tried also to use Cauchy-Schwarz, but without success.



Also, I think the BW (see here https://math.stackexchange.com/tags/buffalo-way/info I tryed!) does not help.










share|cite|improve this question




















  • 2




    Is the function $f : (a,b,c) rightarrow sumlimits_{cyc} frac{6a^3}{13a^2+5b^2} $ concave? Also, C-S = "Cauchy-Schwarz inequality"? Avoid this kind of unconventional abbreviation, please.
    – Vincent
    May 8 '16 at 18:17












  • See also math.stackexchange.com/questions/1775572/…
    – David Quinn
    May 8 '16 at 18:42






  • 1




    @Santropedro There is the following known inequality $sumlimits_{cyc}frac{a^3}{a^2+b^2}geqfrac{a+b+c}{2}$. I think, it's just interesting, for which maximal value of $k$ the following inequality is still true. $sumlimits_{cyc}frac{a^3}{ka^2+b^2}geqfrac{a+b+c}{k+1}$.
    – Michael Rozenberg
    Apr 19 '17 at 17:28






  • 1




    @Santropedro The value $,k=2.6,$ in the OP is pretty sharp. I found that $k=2.603279$ is already beyond: Then Michael's given $(0.785, 1.25, 1.861)$ violates the ineq..
    – Hanno
    May 11 '17 at 18:24






  • 1




    Further result obtained via random-based simulation: $(0.785, 1.2535, 1.873)$ doesn't satisfy the inequality if $,k=2.603262,$. Below that $k$-value I didn't detect a violation.
    – Hanno
    May 11 '17 at 18:50













up vote
25
down vote

favorite
24









up vote
25
down vote

favorite
24






24






Let $a$, $b$ and $c$ be positive numbers. Prove that:
$$frac{a^3}{13a^2+5b^2}+frac{b^3}{13b^2+5c^2}+frac{c^3}{13c^2+5a^2}geqfrac{a+b+c}{18}$$




This inequality is strengthening of the following Vasile Cirtoaje's one, which he created in 2005.




Let $a$, $b$ and $c$ be positive numbers. Prove that:
$$frac{a^3}{2a^2+b^2}+frac{b^3}{2b^2+c^2}+frac{c^3}{2c^2+a^2}geqfrac{a+b+c}{3}.$$




My proof of this inequality you can see here: https://artofproblemsolving.com/community/c6h22937p427220



But this way does not help for the starting inequality.



A big problem we have around the point $(a,b,c)=(0.785, 1.25, 1.861)$ because the difference between the LHS and the RHS in this point is $0.0000158...$.



I tried also to use Cauchy-Schwarz, but without success.



Also, I think the BW (see here https://math.stackexchange.com/tags/buffalo-way/info I tryed!) does not help.










share|cite|improve this question
















Let $a$, $b$ and $c$ be positive numbers. Prove that:
$$frac{a^3}{13a^2+5b^2}+frac{b^3}{13b^2+5c^2}+frac{c^3}{13c^2+5a^2}geqfrac{a+b+c}{18}$$




This inequality is strengthening of the following Vasile Cirtoaje's one, which he created in 2005.




Let $a$, $b$ and $c$ be positive numbers. Prove that:
$$frac{a^3}{2a^2+b^2}+frac{b^3}{2b^2+c^2}+frac{c^3}{2c^2+a^2}geqfrac{a+b+c}{3}.$$




My proof of this inequality you can see here: https://artofproblemsolving.com/community/c6h22937p427220



But this way does not help for the starting inequality.



A big problem we have around the point $(a,b,c)=(0.785, 1.25, 1.861)$ because the difference between the LHS and the RHS in this point is $0.0000158...$.



I tried also to use Cauchy-Schwarz, but without success.



Also, I think the BW (see here https://math.stackexchange.com/tags/buffalo-way/info I tryed!) does not help.







real-analysis inequality contest-math






share|cite|improve this question















share|cite|improve this question













share|cite|improve this question




share|cite|improve this question








edited Dec 8 at 12:34

























asked May 8 '16 at 18:10









Michael Rozenberg

94.9k1588183




94.9k1588183








  • 2




    Is the function $f : (a,b,c) rightarrow sumlimits_{cyc} frac{6a^3}{13a^2+5b^2} $ concave? Also, C-S = "Cauchy-Schwarz inequality"? Avoid this kind of unconventional abbreviation, please.
    – Vincent
    May 8 '16 at 18:17












  • See also math.stackexchange.com/questions/1775572/…
    – David Quinn
    May 8 '16 at 18:42






  • 1




    @Santropedro There is the following known inequality $sumlimits_{cyc}frac{a^3}{a^2+b^2}geqfrac{a+b+c}{2}$. I think, it's just interesting, for which maximal value of $k$ the following inequality is still true. $sumlimits_{cyc}frac{a^3}{ka^2+b^2}geqfrac{a+b+c}{k+1}$.
    – Michael Rozenberg
    Apr 19 '17 at 17:28






  • 1




    @Santropedro The value $,k=2.6,$ in the OP is pretty sharp. I found that $k=2.603279$ is already beyond: Then Michael's given $(0.785, 1.25, 1.861)$ violates the ineq..
    – Hanno
    May 11 '17 at 18:24






  • 1




    Further result obtained via random-based simulation: $(0.785, 1.2535, 1.873)$ doesn't satisfy the inequality if $,k=2.603262,$. Below that $k$-value I didn't detect a violation.
    – Hanno
    May 11 '17 at 18:50














  • 2




    Is the function $f : (a,b,c) rightarrow sumlimits_{cyc} frac{6a^3}{13a^2+5b^2} $ concave? Also, C-S = "Cauchy-Schwarz inequality"? Avoid this kind of unconventional abbreviation, please.
    – Vincent
    May 8 '16 at 18:17












  • See also math.stackexchange.com/questions/1775572/…
    – David Quinn
    May 8 '16 at 18:42






  • 1




    @Santropedro There is the following known inequality $sumlimits_{cyc}frac{a^3}{a^2+b^2}geqfrac{a+b+c}{2}$. I think, it's just interesting, for which maximal value of $k$ the following inequality is still true. $sumlimits_{cyc}frac{a^3}{ka^2+b^2}geqfrac{a+b+c}{k+1}$.
    – Michael Rozenberg
    Apr 19 '17 at 17:28






  • 1




    @Santropedro The value $,k=2.6,$ in the OP is pretty sharp. I found that $k=2.603279$ is already beyond: Then Michael's given $(0.785, 1.25, 1.861)$ violates the ineq..
    – Hanno
    May 11 '17 at 18:24






  • 1




    Further result obtained via random-based simulation: $(0.785, 1.2535, 1.873)$ doesn't satisfy the inequality if $,k=2.603262,$. Below that $k$-value I didn't detect a violation.
    – Hanno
    May 11 '17 at 18:50








2




2




Is the function $f : (a,b,c) rightarrow sumlimits_{cyc} frac{6a^3}{13a^2+5b^2} $ concave? Also, C-S = "Cauchy-Schwarz inequality"? Avoid this kind of unconventional abbreviation, please.
– Vincent
May 8 '16 at 18:17






Is the function $f : (a,b,c) rightarrow sumlimits_{cyc} frac{6a^3}{13a^2+5b^2} $ concave? Also, C-S = "Cauchy-Schwarz inequality"? Avoid this kind of unconventional abbreviation, please.
– Vincent
May 8 '16 at 18:17














See also math.stackexchange.com/questions/1775572/…
– David Quinn
May 8 '16 at 18:42




See also math.stackexchange.com/questions/1775572/…
– David Quinn
May 8 '16 at 18:42




1




1




@Santropedro There is the following known inequality $sumlimits_{cyc}frac{a^3}{a^2+b^2}geqfrac{a+b+c}{2}$. I think, it's just interesting, for which maximal value of $k$ the following inequality is still true. $sumlimits_{cyc}frac{a^3}{ka^2+b^2}geqfrac{a+b+c}{k+1}$.
– Michael Rozenberg
Apr 19 '17 at 17:28




@Santropedro There is the following known inequality $sumlimits_{cyc}frac{a^3}{a^2+b^2}geqfrac{a+b+c}{2}$. I think, it's just interesting, for which maximal value of $k$ the following inequality is still true. $sumlimits_{cyc}frac{a^3}{ka^2+b^2}geqfrac{a+b+c}{k+1}$.
– Michael Rozenberg
Apr 19 '17 at 17:28




1




1




@Santropedro The value $,k=2.6,$ in the OP is pretty sharp. I found that $k=2.603279$ is already beyond: Then Michael's given $(0.785, 1.25, 1.861)$ violates the ineq..
– Hanno
May 11 '17 at 18:24




@Santropedro The value $,k=2.6,$ in the OP is pretty sharp. I found that $k=2.603279$ is already beyond: Then Michael's given $(0.785, 1.25, 1.861)$ violates the ineq..
– Hanno
May 11 '17 at 18:24




1




1




Further result obtained via random-based simulation: $(0.785, 1.2535, 1.873)$ doesn't satisfy the inequality if $,k=2.603262,$. Below that $k$-value I didn't detect a violation.
– Hanno
May 11 '17 at 18:50




Further result obtained via random-based simulation: $(0.785, 1.2535, 1.873)$ doesn't satisfy the inequality if $,k=2.603262,$. Below that $k$-value I didn't detect a violation.
– Hanno
May 11 '17 at 18:50










2 Answers
2






active

oldest

votes

















up vote
3
down vote



+100










Your inequality is equivalent to :
$$sum_{cyc}frac{a}{13}sin(arctan(sqrt{frac{13}{5}}frac{a}{b}))^2geq frac{a+b+c}{18}$$
Each side is divided by $b$, We get:
$$frac{a}{13b} sin(arctan(sqrt{frac{13}{5}}frac{a}{b}))^2+frac{1}{13}sin(arctan(sqrt{frac{13}{5}}frac{b}{c}))^2+frac{c}{13b}sin(arctan(sqrt{frac{13}{5}}frac{c}{a}))^2geq frac{1+frac{a}{b}+frac{c}{b}}{18}$$
Now we put $sqrt{frac{13}{5}}frac{a}{b}=x$, $sqrt{frac{13}{5}}frac{b}{c}=y$, $sqrt{frac{13}{5}}frac{c}{a}=z$, your inequality is equivalent to:
$$sqrt{dfrac{5}{13}}frac{1}{13}dfrac{(x)^3}{(x^2+1)}+dfrac{1}{13}dfrac{(y)^2}{(y^2+1)}+sqrt{dfrac{13}{5}}dfrac{1}{13(y)}dfrac{(z)^2}{(z^2+1)}$$$$geq dfrac{1+sqrt{dfrac{5}{13}}x+sqrt{dfrac{13}{5}}dfrac{1}{y}}{18}$$
with the condition $xyz=(sqrt{frac{13}{5}})^3$.



We study the following function:
$$f(x)=sqrt{dfrac{5}{13}}frac{1}{13}dfrac{(x)^3}{(x^2+1)}+dfrac{1}{13}dfrac{(y)^2}{(y^2+1)}+sqrt{dfrac{13}{5}}dfrac{1}{13(y)}dfrac{(z)^2}{(z^2+1)}-dfrac{1+sqrt{dfrac{5}{13}}x+sqrt{dfrac{13}{5}}dfrac{1}{y}}{18}$$
This function is easily differentiable and the minimum is for $x=sqrt{frac{3sqrt{77}}{17}-frac{26}{17}}=alpha$. So with the condition $xyz=(sqrt{frac{13}{5}})^3$ becomes $yz=frac{(sqrt{frac{13}{5}})^3}{alpha}=beta$. So we have this inequality just with $y$:
$$sqrt{dfrac{5}{13}}frac{1}{13}dfrac{(alpha)^3}{(alpha^2+1)}+dfrac{1}{13}dfrac{(y)^2}{(y^2+1)}+sqrt{dfrac{13}{5}}dfrac{1}{13(y)}dfrac{(frac{beta}{y})^2}{((frac{beta}{y})^2+1)}geqdfrac{1+sqrt{dfrac{5}{13}}alpha+sqrt{dfrac{13}{5}}dfrac{1}{y}}{18}$$
which is easily analyzable. Done!






share|cite|improve this answer























  • Why the last inequality is true?
    – Michael Rozenberg
    Oct 21 '17 at 14:43












  • Wow you analyze rapidly (without irony).I check it with wolfram alpha but I will create a question for this I think .
    – max8128
    Oct 21 '17 at 14:45










  • We can check the starting inequality by WA without your previous work. But maybe WA is wrong (by the way, it happens sometimes!).
    – Michael Rozenberg
    Oct 21 '17 at 15:02






  • 1




    Delete please your "solution". I think it's not good, which you are doing.
    – Michael Rozenberg
    Oct 21 '17 at 16:59






  • 4




    I think it's nothing.
    – Michael Rozenberg
    Oct 25 '17 at 11:15


















up vote
1
down vote













I have finally found a solution . In fact we start to study the 2 variables version of this inequality we have :




$$frac{a^3}{13a^2+5b^2}+frac{b^3}{13b^2+5a^2}geq frac{a+b}{18}$$




Proof:



We have with $x=frac{a}{b}$ :
$$frac{x^3}{13x^2+5}+frac{1}{13+5x^2}geq frac{1+x}{18}$$
Or
$$5(x+1)(x-1)^2(5x^2-8x+5)geq 0$$



So we have (if we permute the variables $a,b,c$ and addition the three inequalities ) :



$$sum_{cyc}frac{a^3}{13a^2+5b^2}+sum_{cyc}frac{a^3}{13a^2+5c^2}geq frac{a+b+c}{9}$$



If we have $sum_{cyc}frac{a^3}{13a^2+5b^2}geqsum_{cyc}frac{a^3}{13a^2+5c^2}$



We have :
$$sum_{cyc}frac{a^3}{13a^2+5b^2}geq frac{a+b+c}{18}$$
But also
$$frac{(a-epsilon)^3}{13(a-epsilon)^2+5b^2}+frac{(b)^3}{13(b)^2+5(c+epsilon)^2}+frac{(c+epsilon)^3}{13(c+epsilon)^2+5(a-epsilon)^2}geq frac{a+b+c}{18}$$
If we put $ageq c $ and $epsilon=a-c$



We finally obtain :
$$sum_{cyc}frac{a^3}{13a^2+5c^2}geq frac{a+b+c}{18}$$



So all the cases are here so it's proved !






share|cite|improve this answer























  • If you'll say that it's proved it's not says that it's indeed proved. Stop please to post a wrong solutions.
    – Michael Rozenberg
    Dec 7 at 15:38










  • Now it's right and proved .
    – max8128
    Dec 8 at 11:07











Your Answer





StackExchange.ifUsing("editor", function () {
return StackExchange.using("mathjaxEditing", function () {
StackExchange.MarkdownEditor.creationCallbacks.add(function (editor, postfix) {
StackExchange.mathjaxEditing.prepareWmdForMathJax(editor, postfix, [["$", "$"], ["\\(","\\)"]]);
});
});
}, "mathjax-editing");

StackExchange.ready(function() {
var channelOptions = {
tags: "".split(" "),
id: "69"
};
initTagRenderer("".split(" "), "".split(" "), channelOptions);

StackExchange.using("externalEditor", function() {
// Have to fire editor after snippets, if snippets enabled
if (StackExchange.settings.snippets.snippetsEnabled) {
StackExchange.using("snippets", function() {
createEditor();
});
}
else {
createEditor();
}
});

function createEditor() {
StackExchange.prepareEditor({
heartbeatType: 'answer',
convertImagesToLinks: true,
noModals: true,
showLowRepImageUploadWarning: true,
reputationToPostImages: 10,
bindNavPrevention: true,
postfix: "",
imageUploader: {
brandingHtml: "Powered by u003ca class="icon-imgur-white" href="https://imgur.com/"u003eu003c/au003e",
contentPolicyHtml: "User contributions licensed under u003ca href="https://creativecommons.org/licenses/by-sa/3.0/"u003ecc by-sa 3.0 with attribution requiredu003c/au003e u003ca href="https://stackoverflow.com/legal/content-policy"u003e(content policy)u003c/au003e",
allowUrls: true
},
noCode: true, onDemand: true,
discardSelector: ".discard-answer"
,immediatelyShowMarkdownHelp:true
});


}
});














draft saved

draft discarded


















StackExchange.ready(
function () {
StackExchange.openid.initPostLogin('.new-post-login', 'https%3a%2f%2fmath.stackexchange.com%2fquestions%2f1777075%2finequality-sum-limits-cyc-fraca313a25b2-geq-fracabc18%23new-answer', 'question_page');
}
);

Post as a guest















Required, but never shown

























2 Answers
2






active

oldest

votes








2 Answers
2






active

oldest

votes









active

oldest

votes






active

oldest

votes








up vote
3
down vote



+100










Your inequality is equivalent to :
$$sum_{cyc}frac{a}{13}sin(arctan(sqrt{frac{13}{5}}frac{a}{b}))^2geq frac{a+b+c}{18}$$
Each side is divided by $b$, We get:
$$frac{a}{13b} sin(arctan(sqrt{frac{13}{5}}frac{a}{b}))^2+frac{1}{13}sin(arctan(sqrt{frac{13}{5}}frac{b}{c}))^2+frac{c}{13b}sin(arctan(sqrt{frac{13}{5}}frac{c}{a}))^2geq frac{1+frac{a}{b}+frac{c}{b}}{18}$$
Now we put $sqrt{frac{13}{5}}frac{a}{b}=x$, $sqrt{frac{13}{5}}frac{b}{c}=y$, $sqrt{frac{13}{5}}frac{c}{a}=z$, your inequality is equivalent to:
$$sqrt{dfrac{5}{13}}frac{1}{13}dfrac{(x)^3}{(x^2+1)}+dfrac{1}{13}dfrac{(y)^2}{(y^2+1)}+sqrt{dfrac{13}{5}}dfrac{1}{13(y)}dfrac{(z)^2}{(z^2+1)}$$$$geq dfrac{1+sqrt{dfrac{5}{13}}x+sqrt{dfrac{13}{5}}dfrac{1}{y}}{18}$$
with the condition $xyz=(sqrt{frac{13}{5}})^3$.



We study the following function:
$$f(x)=sqrt{dfrac{5}{13}}frac{1}{13}dfrac{(x)^3}{(x^2+1)}+dfrac{1}{13}dfrac{(y)^2}{(y^2+1)}+sqrt{dfrac{13}{5}}dfrac{1}{13(y)}dfrac{(z)^2}{(z^2+1)}-dfrac{1+sqrt{dfrac{5}{13}}x+sqrt{dfrac{13}{5}}dfrac{1}{y}}{18}$$
This function is easily differentiable and the minimum is for $x=sqrt{frac{3sqrt{77}}{17}-frac{26}{17}}=alpha$. So with the condition $xyz=(sqrt{frac{13}{5}})^3$ becomes $yz=frac{(sqrt{frac{13}{5}})^3}{alpha}=beta$. So we have this inequality just with $y$:
$$sqrt{dfrac{5}{13}}frac{1}{13}dfrac{(alpha)^3}{(alpha^2+1)}+dfrac{1}{13}dfrac{(y)^2}{(y^2+1)}+sqrt{dfrac{13}{5}}dfrac{1}{13(y)}dfrac{(frac{beta}{y})^2}{((frac{beta}{y})^2+1)}geqdfrac{1+sqrt{dfrac{5}{13}}alpha+sqrt{dfrac{13}{5}}dfrac{1}{y}}{18}$$
which is easily analyzable. Done!






share|cite|improve this answer























  • Why the last inequality is true?
    – Michael Rozenberg
    Oct 21 '17 at 14:43












  • Wow you analyze rapidly (without irony).I check it with wolfram alpha but I will create a question for this I think .
    – max8128
    Oct 21 '17 at 14:45










  • We can check the starting inequality by WA without your previous work. But maybe WA is wrong (by the way, it happens sometimes!).
    – Michael Rozenberg
    Oct 21 '17 at 15:02






  • 1




    Delete please your "solution". I think it's not good, which you are doing.
    – Michael Rozenberg
    Oct 21 '17 at 16:59






  • 4




    I think it's nothing.
    – Michael Rozenberg
    Oct 25 '17 at 11:15















up vote
3
down vote



+100










Your inequality is equivalent to :
$$sum_{cyc}frac{a}{13}sin(arctan(sqrt{frac{13}{5}}frac{a}{b}))^2geq frac{a+b+c}{18}$$
Each side is divided by $b$, We get:
$$frac{a}{13b} sin(arctan(sqrt{frac{13}{5}}frac{a}{b}))^2+frac{1}{13}sin(arctan(sqrt{frac{13}{5}}frac{b}{c}))^2+frac{c}{13b}sin(arctan(sqrt{frac{13}{5}}frac{c}{a}))^2geq frac{1+frac{a}{b}+frac{c}{b}}{18}$$
Now we put $sqrt{frac{13}{5}}frac{a}{b}=x$, $sqrt{frac{13}{5}}frac{b}{c}=y$, $sqrt{frac{13}{5}}frac{c}{a}=z$, your inequality is equivalent to:
$$sqrt{dfrac{5}{13}}frac{1}{13}dfrac{(x)^3}{(x^2+1)}+dfrac{1}{13}dfrac{(y)^2}{(y^2+1)}+sqrt{dfrac{13}{5}}dfrac{1}{13(y)}dfrac{(z)^2}{(z^2+1)}$$$$geq dfrac{1+sqrt{dfrac{5}{13}}x+sqrt{dfrac{13}{5}}dfrac{1}{y}}{18}$$
with the condition $xyz=(sqrt{frac{13}{5}})^3$.



We study the following function:
$$f(x)=sqrt{dfrac{5}{13}}frac{1}{13}dfrac{(x)^3}{(x^2+1)}+dfrac{1}{13}dfrac{(y)^2}{(y^2+1)}+sqrt{dfrac{13}{5}}dfrac{1}{13(y)}dfrac{(z)^2}{(z^2+1)}-dfrac{1+sqrt{dfrac{5}{13}}x+sqrt{dfrac{13}{5}}dfrac{1}{y}}{18}$$
This function is easily differentiable and the minimum is for $x=sqrt{frac{3sqrt{77}}{17}-frac{26}{17}}=alpha$. So with the condition $xyz=(sqrt{frac{13}{5}})^3$ becomes $yz=frac{(sqrt{frac{13}{5}})^3}{alpha}=beta$. So we have this inequality just with $y$:
$$sqrt{dfrac{5}{13}}frac{1}{13}dfrac{(alpha)^3}{(alpha^2+1)}+dfrac{1}{13}dfrac{(y)^2}{(y^2+1)}+sqrt{dfrac{13}{5}}dfrac{1}{13(y)}dfrac{(frac{beta}{y})^2}{((frac{beta}{y})^2+1)}geqdfrac{1+sqrt{dfrac{5}{13}}alpha+sqrt{dfrac{13}{5}}dfrac{1}{y}}{18}$$
which is easily analyzable. Done!






share|cite|improve this answer























  • Why the last inequality is true?
    – Michael Rozenberg
    Oct 21 '17 at 14:43












  • Wow you analyze rapidly (without irony).I check it with wolfram alpha but I will create a question for this I think .
    – max8128
    Oct 21 '17 at 14:45










  • We can check the starting inequality by WA without your previous work. But maybe WA is wrong (by the way, it happens sometimes!).
    – Michael Rozenberg
    Oct 21 '17 at 15:02






  • 1




    Delete please your "solution". I think it's not good, which you are doing.
    – Michael Rozenberg
    Oct 21 '17 at 16:59






  • 4




    I think it's nothing.
    – Michael Rozenberg
    Oct 25 '17 at 11:15













up vote
3
down vote



+100







up vote
3
down vote



+100




+100




Your inequality is equivalent to :
$$sum_{cyc}frac{a}{13}sin(arctan(sqrt{frac{13}{5}}frac{a}{b}))^2geq frac{a+b+c}{18}$$
Each side is divided by $b$, We get:
$$frac{a}{13b} sin(arctan(sqrt{frac{13}{5}}frac{a}{b}))^2+frac{1}{13}sin(arctan(sqrt{frac{13}{5}}frac{b}{c}))^2+frac{c}{13b}sin(arctan(sqrt{frac{13}{5}}frac{c}{a}))^2geq frac{1+frac{a}{b}+frac{c}{b}}{18}$$
Now we put $sqrt{frac{13}{5}}frac{a}{b}=x$, $sqrt{frac{13}{5}}frac{b}{c}=y$, $sqrt{frac{13}{5}}frac{c}{a}=z$, your inequality is equivalent to:
$$sqrt{dfrac{5}{13}}frac{1}{13}dfrac{(x)^3}{(x^2+1)}+dfrac{1}{13}dfrac{(y)^2}{(y^2+1)}+sqrt{dfrac{13}{5}}dfrac{1}{13(y)}dfrac{(z)^2}{(z^2+1)}$$$$geq dfrac{1+sqrt{dfrac{5}{13}}x+sqrt{dfrac{13}{5}}dfrac{1}{y}}{18}$$
with the condition $xyz=(sqrt{frac{13}{5}})^3$.



We study the following function:
$$f(x)=sqrt{dfrac{5}{13}}frac{1}{13}dfrac{(x)^3}{(x^2+1)}+dfrac{1}{13}dfrac{(y)^2}{(y^2+1)}+sqrt{dfrac{13}{5}}dfrac{1}{13(y)}dfrac{(z)^2}{(z^2+1)}-dfrac{1+sqrt{dfrac{5}{13}}x+sqrt{dfrac{13}{5}}dfrac{1}{y}}{18}$$
This function is easily differentiable and the minimum is for $x=sqrt{frac{3sqrt{77}}{17}-frac{26}{17}}=alpha$. So with the condition $xyz=(sqrt{frac{13}{5}})^3$ becomes $yz=frac{(sqrt{frac{13}{5}})^3}{alpha}=beta$. So we have this inequality just with $y$:
$$sqrt{dfrac{5}{13}}frac{1}{13}dfrac{(alpha)^3}{(alpha^2+1)}+dfrac{1}{13}dfrac{(y)^2}{(y^2+1)}+sqrt{dfrac{13}{5}}dfrac{1}{13(y)}dfrac{(frac{beta}{y})^2}{((frac{beta}{y})^2+1)}geqdfrac{1+sqrt{dfrac{5}{13}}alpha+sqrt{dfrac{13}{5}}dfrac{1}{y}}{18}$$
which is easily analyzable. Done!






share|cite|improve this answer














Your inequality is equivalent to :
$$sum_{cyc}frac{a}{13}sin(arctan(sqrt{frac{13}{5}}frac{a}{b}))^2geq frac{a+b+c}{18}$$
Each side is divided by $b$, We get:
$$frac{a}{13b} sin(arctan(sqrt{frac{13}{5}}frac{a}{b}))^2+frac{1}{13}sin(arctan(sqrt{frac{13}{5}}frac{b}{c}))^2+frac{c}{13b}sin(arctan(sqrt{frac{13}{5}}frac{c}{a}))^2geq frac{1+frac{a}{b}+frac{c}{b}}{18}$$
Now we put $sqrt{frac{13}{5}}frac{a}{b}=x$, $sqrt{frac{13}{5}}frac{b}{c}=y$, $sqrt{frac{13}{5}}frac{c}{a}=z$, your inequality is equivalent to:
$$sqrt{dfrac{5}{13}}frac{1}{13}dfrac{(x)^3}{(x^2+1)}+dfrac{1}{13}dfrac{(y)^2}{(y^2+1)}+sqrt{dfrac{13}{5}}dfrac{1}{13(y)}dfrac{(z)^2}{(z^2+1)}$$$$geq dfrac{1+sqrt{dfrac{5}{13}}x+sqrt{dfrac{13}{5}}dfrac{1}{y}}{18}$$
with the condition $xyz=(sqrt{frac{13}{5}})^3$.



We study the following function:
$$f(x)=sqrt{dfrac{5}{13}}frac{1}{13}dfrac{(x)^3}{(x^2+1)}+dfrac{1}{13}dfrac{(y)^2}{(y^2+1)}+sqrt{dfrac{13}{5}}dfrac{1}{13(y)}dfrac{(z)^2}{(z^2+1)}-dfrac{1+sqrt{dfrac{5}{13}}x+sqrt{dfrac{13}{5}}dfrac{1}{y}}{18}$$
This function is easily differentiable and the minimum is for $x=sqrt{frac{3sqrt{77}}{17}-frac{26}{17}}=alpha$. So with the condition $xyz=(sqrt{frac{13}{5}})^3$ becomes $yz=frac{(sqrt{frac{13}{5}})^3}{alpha}=beta$. So we have this inequality just with $y$:
$$sqrt{dfrac{5}{13}}frac{1}{13}dfrac{(alpha)^3}{(alpha^2+1)}+dfrac{1}{13}dfrac{(y)^2}{(y^2+1)}+sqrt{dfrac{13}{5}}dfrac{1}{13(y)}dfrac{(frac{beta}{y})^2}{((frac{beta}{y})^2+1)}geqdfrac{1+sqrt{dfrac{5}{13}}alpha+sqrt{dfrac{13}{5}}dfrac{1}{y}}{18}$$
which is easily analyzable. Done!







share|cite|improve this answer














share|cite|improve this answer



share|cite|improve this answer








edited Dec 8 at 11:14









user302797

19.7k92252




19.7k92252










answered Oct 21 '17 at 14:41









max8128

1,033421




1,033421












  • Why the last inequality is true?
    – Michael Rozenberg
    Oct 21 '17 at 14:43












  • Wow you analyze rapidly (without irony).I check it with wolfram alpha but I will create a question for this I think .
    – max8128
    Oct 21 '17 at 14:45










  • We can check the starting inequality by WA without your previous work. But maybe WA is wrong (by the way, it happens sometimes!).
    – Michael Rozenberg
    Oct 21 '17 at 15:02






  • 1




    Delete please your "solution". I think it's not good, which you are doing.
    – Michael Rozenberg
    Oct 21 '17 at 16:59






  • 4




    I think it's nothing.
    – Michael Rozenberg
    Oct 25 '17 at 11:15


















  • Why the last inequality is true?
    – Michael Rozenberg
    Oct 21 '17 at 14:43












  • Wow you analyze rapidly (without irony).I check it with wolfram alpha but I will create a question for this I think .
    – max8128
    Oct 21 '17 at 14:45










  • We can check the starting inequality by WA without your previous work. But maybe WA is wrong (by the way, it happens sometimes!).
    – Michael Rozenberg
    Oct 21 '17 at 15:02






  • 1




    Delete please your "solution". I think it's not good, which you are doing.
    – Michael Rozenberg
    Oct 21 '17 at 16:59






  • 4




    I think it's nothing.
    – Michael Rozenberg
    Oct 25 '17 at 11:15
















Why the last inequality is true?
– Michael Rozenberg
Oct 21 '17 at 14:43






Why the last inequality is true?
– Michael Rozenberg
Oct 21 '17 at 14:43














Wow you analyze rapidly (without irony).I check it with wolfram alpha but I will create a question for this I think .
– max8128
Oct 21 '17 at 14:45




Wow you analyze rapidly (without irony).I check it with wolfram alpha but I will create a question for this I think .
– max8128
Oct 21 '17 at 14:45












We can check the starting inequality by WA without your previous work. But maybe WA is wrong (by the way, it happens sometimes!).
– Michael Rozenberg
Oct 21 '17 at 15:02




We can check the starting inequality by WA without your previous work. But maybe WA is wrong (by the way, it happens sometimes!).
– Michael Rozenberg
Oct 21 '17 at 15:02




1




1




Delete please your "solution". I think it's not good, which you are doing.
– Michael Rozenberg
Oct 21 '17 at 16:59




Delete please your "solution". I think it's not good, which you are doing.
– Michael Rozenberg
Oct 21 '17 at 16:59




4




4




I think it's nothing.
– Michael Rozenberg
Oct 25 '17 at 11:15




I think it's nothing.
– Michael Rozenberg
Oct 25 '17 at 11:15










up vote
1
down vote













I have finally found a solution . In fact we start to study the 2 variables version of this inequality we have :




$$frac{a^3}{13a^2+5b^2}+frac{b^3}{13b^2+5a^2}geq frac{a+b}{18}$$




Proof:



We have with $x=frac{a}{b}$ :
$$frac{x^3}{13x^2+5}+frac{1}{13+5x^2}geq frac{1+x}{18}$$
Or
$$5(x+1)(x-1)^2(5x^2-8x+5)geq 0$$



So we have (if we permute the variables $a,b,c$ and addition the three inequalities ) :



$$sum_{cyc}frac{a^3}{13a^2+5b^2}+sum_{cyc}frac{a^3}{13a^2+5c^2}geq frac{a+b+c}{9}$$



If we have $sum_{cyc}frac{a^3}{13a^2+5b^2}geqsum_{cyc}frac{a^3}{13a^2+5c^2}$



We have :
$$sum_{cyc}frac{a^3}{13a^2+5b^2}geq frac{a+b+c}{18}$$
But also
$$frac{(a-epsilon)^3}{13(a-epsilon)^2+5b^2}+frac{(b)^3}{13(b)^2+5(c+epsilon)^2}+frac{(c+epsilon)^3}{13(c+epsilon)^2+5(a-epsilon)^2}geq frac{a+b+c}{18}$$
If we put $ageq c $ and $epsilon=a-c$



We finally obtain :
$$sum_{cyc}frac{a^3}{13a^2+5c^2}geq frac{a+b+c}{18}$$



So all the cases are here so it's proved !






share|cite|improve this answer























  • If you'll say that it's proved it's not says that it's indeed proved. Stop please to post a wrong solutions.
    – Michael Rozenberg
    Dec 7 at 15:38










  • Now it's right and proved .
    – max8128
    Dec 8 at 11:07















up vote
1
down vote













I have finally found a solution . In fact we start to study the 2 variables version of this inequality we have :




$$frac{a^3}{13a^2+5b^2}+frac{b^3}{13b^2+5a^2}geq frac{a+b}{18}$$




Proof:



We have with $x=frac{a}{b}$ :
$$frac{x^3}{13x^2+5}+frac{1}{13+5x^2}geq frac{1+x}{18}$$
Or
$$5(x+1)(x-1)^2(5x^2-8x+5)geq 0$$



So we have (if we permute the variables $a,b,c$ and addition the three inequalities ) :



$$sum_{cyc}frac{a^3}{13a^2+5b^2}+sum_{cyc}frac{a^3}{13a^2+5c^2}geq frac{a+b+c}{9}$$



If we have $sum_{cyc}frac{a^3}{13a^2+5b^2}geqsum_{cyc}frac{a^3}{13a^2+5c^2}$



We have :
$$sum_{cyc}frac{a^3}{13a^2+5b^2}geq frac{a+b+c}{18}$$
But also
$$frac{(a-epsilon)^3}{13(a-epsilon)^2+5b^2}+frac{(b)^3}{13(b)^2+5(c+epsilon)^2}+frac{(c+epsilon)^3}{13(c+epsilon)^2+5(a-epsilon)^2}geq frac{a+b+c}{18}$$
If we put $ageq c $ and $epsilon=a-c$



We finally obtain :
$$sum_{cyc}frac{a^3}{13a^2+5c^2}geq frac{a+b+c}{18}$$



So all the cases are here so it's proved !






share|cite|improve this answer























  • If you'll say that it's proved it's not says that it's indeed proved. Stop please to post a wrong solutions.
    – Michael Rozenberg
    Dec 7 at 15:38










  • Now it's right and proved .
    – max8128
    Dec 8 at 11:07













up vote
1
down vote










up vote
1
down vote









I have finally found a solution . In fact we start to study the 2 variables version of this inequality we have :




$$frac{a^3}{13a^2+5b^2}+frac{b^3}{13b^2+5a^2}geq frac{a+b}{18}$$




Proof:



We have with $x=frac{a}{b}$ :
$$frac{x^3}{13x^2+5}+frac{1}{13+5x^2}geq frac{1+x}{18}$$
Or
$$5(x+1)(x-1)^2(5x^2-8x+5)geq 0$$



So we have (if we permute the variables $a,b,c$ and addition the three inequalities ) :



$$sum_{cyc}frac{a^3}{13a^2+5b^2}+sum_{cyc}frac{a^3}{13a^2+5c^2}geq frac{a+b+c}{9}$$



If we have $sum_{cyc}frac{a^3}{13a^2+5b^2}geqsum_{cyc}frac{a^3}{13a^2+5c^2}$



We have :
$$sum_{cyc}frac{a^3}{13a^2+5b^2}geq frac{a+b+c}{18}$$
But also
$$frac{(a-epsilon)^3}{13(a-epsilon)^2+5b^2}+frac{(b)^3}{13(b)^2+5(c+epsilon)^2}+frac{(c+epsilon)^3}{13(c+epsilon)^2+5(a-epsilon)^2}geq frac{a+b+c}{18}$$
If we put $ageq c $ and $epsilon=a-c$



We finally obtain :
$$sum_{cyc}frac{a^3}{13a^2+5c^2}geq frac{a+b+c}{18}$$



So all the cases are here so it's proved !






share|cite|improve this answer














I have finally found a solution . In fact we start to study the 2 variables version of this inequality we have :




$$frac{a^3}{13a^2+5b^2}+frac{b^3}{13b^2+5a^2}geq frac{a+b}{18}$$




Proof:



We have with $x=frac{a}{b}$ :
$$frac{x^3}{13x^2+5}+frac{1}{13+5x^2}geq frac{1+x}{18}$$
Or
$$5(x+1)(x-1)^2(5x^2-8x+5)geq 0$$



So we have (if we permute the variables $a,b,c$ and addition the three inequalities ) :



$$sum_{cyc}frac{a^3}{13a^2+5b^2}+sum_{cyc}frac{a^3}{13a^2+5c^2}geq frac{a+b+c}{9}$$



If we have $sum_{cyc}frac{a^3}{13a^2+5b^2}geqsum_{cyc}frac{a^3}{13a^2+5c^2}$



We have :
$$sum_{cyc}frac{a^3}{13a^2+5b^2}geq frac{a+b+c}{18}$$
But also
$$frac{(a-epsilon)^3}{13(a-epsilon)^2+5b^2}+frac{(b)^3}{13(b)^2+5(c+epsilon)^2}+frac{(c+epsilon)^3}{13(c+epsilon)^2+5(a-epsilon)^2}geq frac{a+b+c}{18}$$
If we put $ageq c $ and $epsilon=a-c$



We finally obtain :
$$sum_{cyc}frac{a^3}{13a^2+5c^2}geq frac{a+b+c}{18}$$



So all the cases are here so it's proved !







share|cite|improve this answer














share|cite|improve this answer



share|cite|improve this answer








edited Dec 8 at 11:07

























answered Dec 7 at 14:00









max8128

1,033421




1,033421












  • If you'll say that it's proved it's not says that it's indeed proved. Stop please to post a wrong solutions.
    – Michael Rozenberg
    Dec 7 at 15:38










  • Now it's right and proved .
    – max8128
    Dec 8 at 11:07


















  • If you'll say that it's proved it's not says that it's indeed proved. Stop please to post a wrong solutions.
    – Michael Rozenberg
    Dec 7 at 15:38










  • Now it's right and proved .
    – max8128
    Dec 8 at 11:07
















If you'll say that it's proved it's not says that it's indeed proved. Stop please to post a wrong solutions.
– Michael Rozenberg
Dec 7 at 15:38




If you'll say that it's proved it's not says that it's indeed proved. Stop please to post a wrong solutions.
– Michael Rozenberg
Dec 7 at 15:38












Now it's right and proved .
– max8128
Dec 8 at 11:07




Now it's right and proved .
– max8128
Dec 8 at 11:07


















draft saved

draft discarded




















































Thanks for contributing an answer to Mathematics Stack Exchange!


  • Please be sure to answer the question. Provide details and share your research!

But avoid



  • Asking for help, clarification, or responding to other answers.

  • Making statements based on opinion; back them up with references or personal experience.


Use MathJax to format equations. MathJax reference.


To learn more, see our tips on writing great answers.





Some of your past answers have not been well-received, and you're in danger of being blocked from answering.


Please pay close attention to the following guidance:


  • Please be sure to answer the question. Provide details and share your research!

But avoid



  • Asking for help, clarification, or responding to other answers.

  • Making statements based on opinion; back them up with references or personal experience.


To learn more, see our tips on writing great answers.




draft saved


draft discarded














StackExchange.ready(
function () {
StackExchange.openid.initPostLogin('.new-post-login', 'https%3a%2f%2fmath.stackexchange.com%2fquestions%2f1777075%2finequality-sum-limits-cyc-fraca313a25b2-geq-fracabc18%23new-answer', 'question_page');
}
);

Post as a guest















Required, but never shown





















































Required, but never shown














Required, but never shown












Required, but never shown







Required, but never shown

































Required, but never shown














Required, but never shown












Required, but never shown







Required, but never shown







Popular posts from this blog

Le Mesnil-Réaume

Ida-Boy-Ed-Garten

web3.py web3.isConnected() returns false always